Đến nội dung

Stranger411 nội dung

Có 85 mục bởi Stranger411 (Tìm giới hạn từ 29-04-2020)



Sắp theo                Sắp xếp  

#313754 $$\dfrac{b(a+b)}{(c+a)^2}+\dfrac{c(c+b)}{(a+b)^2}+\d...

Đã gửi bởi Stranger411 on 01-05-2012 - 20:13 trong Bất đẳng thức và cực trị

Bài toán 3.
Cho các số thực dương $a, b, c$ sao cho $abc=1$. Chứng minh rằng :
$$\dfrac{1}{(1+a)^2(b+c)}+\dfrac{1}{(1+b)^2(c+a)}+\dfrac{1}{(1+c)^2(a+b)}\le \dfrac{3}{8}$$


Trần Quốc Anh


Hình đã gửi
Trước tiên, ta chứng minh bổ đề:
$$(a+1)(b+c)\ge \frac{(b+1)(c+1)}{\sqrt{bc}}$$
Có thể dùng Cauchy-Schwwarz để chứng minh hoặc biến đổi tương đương ;)

Bất đẳng thức cần chứng minh trở thành:
$$\sum{\frac{1}{{{(a+1)}^{2}}(b+c)}}\le \sum{\frac{bc}{(a+1)(b+1)(zc+1)}}$$

Vậy, ta chỉ cần chứng minh:
$$(a+1)(b+1)(c+1)\ge \frac{8}{3}(\sqrt{ab}+\sqrt{bc}+\sqrt{ca})$$
Và đây là hệ quả của bđt:

$$(x+y)(y+z)(z+x)\ge \frac{8}{9}(x+y+z)(xy+yz+zx)$$
Bất đẳng thức đã được chứng minh. $\blacksquare$


Ps: anh phuc_90 vào góp vui đi ak :P
Bài 1: Giải bằng SOS mất 3 trang giấy :-ss Mọi người ai có cách giải ngắn hơn ko :-ss



#313293 Balkan MO 2012

Đã gửi bởi Stranger411 on 29-04-2012 - 14:46 trong Thi HSG Quốc gia và Quốc tế

Balkan MO 2012 - 28 April 2012

Bài 2. Prove that \[ \sum_{cyc}(x+y)\sqrt{(z+x)(z+y)}\geq 4(xy+yz+zx), \] for all positive real numbers $x,y$ and $z$.

Tuy là đề thi quốc gia nhưng mình thấy bài này khá lỏng :D
Cách 1:
Áp dụng bđt Cauchy-Schwarz, ta có:


$\left( x+y \right)\sqrt{\left( z+x \right)\left( z+y \right)}\ge \left( x+y \right)\left( z+\sqrt{xy} \right)$

$=\left( x+y \right)z+\left( x+y \right)\sqrt{xy}\ge \left( x+y \right)z+2xy$
Cách 2: Bằng cách dùng bổ đề của huymit_95, ta viết lại bđt như sau:
$\sum {\frac{1}{{\sqrt {\left( {x + y} \right)\left( {x + z} \right)} }}} \ge \frac{{4\left( {xy + yz + zx} \right)}}{{\left( {x + y} \right)\left( {y + z} \right)\left( {z + x} \right)}}$
Ta có: $\left( {x + y} \right)\left( {y + z} \right)\left( {z + x} \right) \ge \frac{8}{9}\left( {xy + yz + zx} \right)\left( {x + y + z} \right)$
Nên ta cần chứng minh: $\sum {\frac{1}{{\sqrt {\left( {x + y} \right)\left( {x + z} \right)} }}} \ge \frac{9}{{2\left( {x + y + z} \right)}}$
Và bđt này hoàn toàn đúng với bđt Cauchy-Schwarz:
$\sum {\frac{1}{{\sqrt {\left( {x + y} \right)\left( {x + z} \right)} }}} \ge \sum {\frac{2}{{x + y + 2z}}} \ge \frac{9}{{2\left( {x + y + z} \right)}}$



#313291 ${{a}^{3}}+{{b}^{3}}+{{c}^{3}}+kabc\ge 3+k$

Đã gửi bởi Stranger411 on 29-04-2012 - 14:32 trong Bất đẳng thức - Cực trị

Mình đang viết 1 chuyên đề về phương pháp tìm hằng số lớn nhất cho bđt.
Mong các bạn góp ý :D

Bài toán 1: Cho các số thực không âm a,b,c thỏa mãn $a+b+c=3$. Tìm hằng số k lớn nhất để bđt sau đúng:

$${{a}^{3}}+{{b}^{3}}+{{c}^{3}}+kabc\ge 3+k$$

Bài toán 2: (Stranger411)Cho các số thực không âm a,b,c thỏa $ab+bc+ca=3$. Tìm hằng số k lớn nhất để bđt sau đúng:

$${{a}^{3}}+{{b}^{3}}+{{c}^{3}}+kabc\ge 3+k$$

Chú ý: Với bài toán 2,ta có một số kết quả:

$k=6$: Hệ quả của bđt Schur
$k=7$: Bài toán đã được arqady giải ở Mathlinks
Tuy nhiên,$k=7$ không phải hằng số tốt nhất cho bđt trên.



#313126 Bài 1. cho $a,b,c>0$có $abc=1$ chứng minh rằng $...

Đã gửi bởi Stranger411 on 28-04-2012 - 14:02 trong Bất đẳng thức - Cực trị

Bài 1. cho $a,b,c>0$có $abc=1$ chứng minh rằng
$\frac{1}{a^3+b+c}+\frac{1}{b^3+a+c}+\frac{1}{c^3+a+b}\leq \frac{3}{a+b+c}$
Bài 2.cho $a,b,c>0 abc=1$ tìm GTNN của
$P=\frac{1}{2a^3+b^3+c^3+2}+\frac{1}{a^3+2b^3+c^3+2}+\frac{1}{a^3+b^3+2c^3+2}$
Bài 3.chứng minh:
$\frac{a^3}{a^2+ab+2b^2}+\frac{b^3}{b^2+bc+2c^2}+\frac{c^3}{c^2+ab+2a^2}\geq \frac{a+b+c}{4}$
Bài 4. $ x+y+z=3$,tìm giá trị nhỏ nhất của biểu thức

$P=\frac{x}{y^3+16}+\frac{y}{z^3+16}+\frac{z}{x^3+16}$

nếu ko có trả lời một tuần nữa minh sẽ post dáp án

Có lẽ cũng chẳng cần post đáp án nữa đâu bạn :lol:
Bài 2:
Áp dụng bđt Cauchy, ta đc:
$$ \frac{1}{2{{a}^{3}}+{{b}^{3}}+{{c}^{3}}+2}\le \frac{1}{4}\left( \frac{1}{{{a}^{3}}+{{b}^{3}}+1}+\frac{1}{{{a}^{3}}+{{c}^{3}}+1} \right) $$
Từ đó, ta được: $ P\le \frac{1}{2}\left( \frac{1}{{{a}^{3}}+{{b}^{3}}+1}+\frac{1}{{{a}^{3}}+{{c}^{3}}+1}+\frac{1}{{{c}^{3}}+{{b}^{3}}+1} \right)\le \frac{1}{2} $

Vì $ {{a}^{3}}+{{b}^{3}}+1\ge ab\left( a+b \right)+1=ab\left( a+b+c \right) $
Bài 3: Sử dụng pp tiếp tuyến, ta chứng minh:

$$ \frac{{{a}^{3}}}{{{a}^{2}}+ab+2{{b}^{2}}}\ge \frac{9a-5b}{16} $$
Cộng các bđt tương tự, ta có đpcm.



#312728 Cho $a,b,c>0$và $ab+bc+ca+abc=4$ .CM$a^3+b^3+c^3...

Đã gửi bởi Stranger411 on 25-04-2012 - 23:19 trong Bất đẳng thức và cực trị

Cho$a,b,c>0$ thỏa mãn $ab+bc+ca+abc=4$.Chứng minh
$a^3+b^3+c^3+9abc\geq 4(a+b+c)$

Phải nói đây một bất đẳng thức khá bá đạo.
Nhưng nếu làm bđt nhiều thì cũng không khó gì để nhận ra nó.
Đổi biến $a=\frac{2x}{y+z},b=\frac{2y}{x+z},c=\frac{2z}{x+y} $, ta được $ab+bc+ca+abc=4$
Lời giải còn lại của bài toán các bạn xem trong file đính kèm.

File gửi kèm




#312558 Bất đẳng thức $4$ biến thoả $ abcd=1$

Đã gửi bởi Stranger411 on 25-04-2012 - 08:17 trong Bất đẳng thức - Cực trị

Cho các số thực dương $a,b,c,d$ thoả mãn: $abcd=1$.Chứng minh rằng:
$$ 3(a^2+b^2+c^2+d^2) + 4 \ge (a+b+c+d)^2$$

Áp dụng bất đẳng thức Tukervici, ta có:
$ 3(a^4+b^4+c^4+d^4) + 4abcd \ge (a^2+b^2+c^2+d^2)^2$
Bất đẳng thức cần chứng minh là dạng tương đương của bất đẳng thức trên. $\blacksquare$
Bất đẳng thức trên dùng pp FMPX để chứng minh.



#312453 Topic: INEQUALITIES (PART II)

Đã gửi bởi Stranger411 on 24-04-2012 - 19:32 trong Bất đẳng thức - Cực trị

Problem 8: Cho a,b,c là các số thực thỏa $a+b+c=3$. Chứng minh rằng:
$$\frac{a^2-bc}{a^2+3}+\frac{b^2-ac}{b^2+3}+\frac{c^2-ab}{c^2+3}\geq 0$$

Bài này khá yếu.
Bằng cách phân tích trực tiếp, ta được:
$$\left( {{a}^{2}}+{{b}^{2}}+{{c}^{2}}-ab-bc-ca \right)\sum{\frac{{{\left( a-b \right)}^{2}}}{\left( {{a}^{2}}+3 \right)\left( {{b}^{2}}+3 \right)}}\ge 0$$
Ta có đpcm. $\blacksquare$

Problem 9: Cho a,b,c là các số thực thỏa $a+b+c=1$. Chứng minh rằng:
$$\frac{\left( {{a}^{2}}+{{b}^{2}} \right)}{{{\left( a+b \right)}^{2}}}\frac{\left( {{c}^{2}}+{{b}^{2}} \right)}{{{\left( c+b \right)}^{2}}}\frac{\left( {{a}^{2}}+{{c}^{2}} \right)}{{{\left( a+c \right)}^{2}}}\ge \frac{3}{8}\left( {{a}^{2}}+{{b}^{2}}+{{c}^{2}} \right)$$



#312448 $\frac{1}{2-\cos A}+\frac{1}{2-\cos B}+\frac{1}...

Đã gửi bởi Stranger411 on 24-04-2012 - 19:17 trong Bất đẳng thức - Cực trị

Hình đã gửiÔng bạn của mình đây mà
Trước hết ta sẽ chứng minh $$cosA+cosB+cosC\geq -\frac{3}{2}$$
Cho $\overrightarrow{OA};\overrightarrow{OB};\overrightarrow{OC}$ là 3 véc tơ đơn vị. ( các góc $A,B,C \in (0; \pi)$)
Ta dễ dàng chứng minh được $(\overrightarrow{OA}+\overrightarrow{OB}+\overrightarrow{OC})^2=3+2(cosA+cosB+cosC) \ge 0$
Từ đây suy ra $$cosA+cosB+cosC\geq \frac{-3}{2}$$
Áp dụng bất đẳng thức AM-HM ta có:
$$\frac{1}{2-cosA}+\frac{1}{2-cosB}+\frac{1}{2-cosC}\geq \frac{9}{3-cosA-cosB-cosC}\geq \frac{9}{3+\frac{3}{2}}=2$$

Một lỗi sai khá cơ bản đó ông bạn Hình đã gửi
$$2\cos (\overrightarrow{OA},\overrightarrow{OB})= \cos2C$$
Từ đó, ta có:
$$cos2A+cos2B+cos2C\geq \frac{-3}{2}$$
Bài này không đơn giản như vậy đâu



#312383 CMR: $xy+yz+zx\leq 8$

Đã gửi bởi Stranger411 on 24-04-2012 - 10:36 trong Bất đẳng thức và cực trị

Cho $x,y,z$ la nghiệm của hệ pt:$\left\{\begin{matrix} x^2+xy+y^2=3\\ y^2+yz+z^2=16\end{matrix}\right.$
CMR: $xy+yz+zx\leq 8$

Áp dụng bất đẳng thức Cauchy-Schwarz, ta có:
${{\left( xy+yz+zx \right)}^{2}}$
$=\left[ x\left( y+\frac{z}{2} \right)+z\left( y+\frac{x}{2} \right) \right]$
$\le \left[ {{x}^{2}}+\frac{4}{3}{{\left( y+\frac{x}{2} \right)}^{2}} \right]\left[ {{\left( y+\frac{z}{2} \right)}^{2}}+\frac{3}{4}{{z}^{2}} \right]$
$=\frac{4}{3}\left( {{x}^{2}}+xy+{{y}^{2}} \right)\left( {{y}^{2}}+yz+{{z}^{2}} \right)=64$
$\Leftrightarrow xy+yz+zx\le 8$ (đpcm)



#312379 $\frac{1}{2-\cos A}+\frac{1}{2-\cos B}+\frac{1}...

Đã gửi bởi Stranger411 on 24-04-2012 - 10:28 trong Bất đẳng thức - Cực trị

Bài 1: Cho $\Delta ABC$. Chứng minh:
$$\frac{1}{2-\cos A}+\frac{1}{2-\cos B}+\frac{1}{2-\cos C}\ge 2$$


Bài 2: Cho $\Delta ABC$. Chứng minh:
$$\frac{\left( 1-\sin \frac{A}{2} \right)\left( 1+\cos \frac{A}{2} \right)}{\sin \frac{A}{2}\left( 1+\sin \frac{A}{2} \right)}+\frac{\left( 1-\sin \frac{B}{2} \right)\left( 1+\cos \frac{B}{2} \right)}{\sin \frac{B}{2}\left( 1+\sin \frac{B}{2} \right)}+\frac{\left( 1-\sin \frac{C}{2} \right)\left( 1+\cos \frac{C}{2} \right)}{\sin \frac{C}{2}\left( 1+\sin \frac{C}{2} \right)}\ge 2+\sqrt{3}$$